If $lim f(x) = 0,$ then $lim 1/f(x) = infty.$












0












$begingroup$



Suppose that $f:DtoBbb R$, where $D$ is a subset of $Bbb R$ and $a$ is an accumulation point of $D$, $lim_{xto a+}f(x)=0$, and $f(x)ne0$ for any $x$ in $D$ in some neighborhood of $a$.
I understand from the following proof that if $lim f(x) = 0,$ then $lim 1/|f(x)| = infty.$ : proof for |f(x)|,
however, does it also apply for $lim 1/f(x)= infty$ ? or $lim 1/f(x)= -infty$ If not then why?











share|cite|improve this question











$endgroup$








  • 3




    $begingroup$
    What if $f$ only takes negative values?
    $endgroup$
    – José Carlos Santos
    Dec 5 '18 at 11:21






  • 2




    $begingroup$
    the problem is whether you consider $infty$ to be either $pm infty$ or only $+infty$
    $endgroup$
    – TheD0ubleT
    Dec 5 '18 at 11:22












  • $begingroup$
    What if $f$ takes both positive and negative values ($f$ would not be continuous)?
    $endgroup$
    – Michael Burr
    Dec 5 '18 at 11:23










  • $begingroup$
    @MichaelBurr how can f take both of those, conjure to 0 and then not be continuous? when it's $lim_{xto a+}$?
    $endgroup$
    – Yuki1112
    Dec 5 '18 at 11:25












  • $begingroup$
    @TheD0ubleT either, sorry, I edited it to be more clear on the matter
    $endgroup$
    – Yuki1112
    Dec 5 '18 at 11:31
















0












$begingroup$



Suppose that $f:DtoBbb R$, where $D$ is a subset of $Bbb R$ and $a$ is an accumulation point of $D$, $lim_{xto a+}f(x)=0$, and $f(x)ne0$ for any $x$ in $D$ in some neighborhood of $a$.
I understand from the following proof that if $lim f(x) = 0,$ then $lim 1/|f(x)| = infty.$ : proof for |f(x)|,
however, does it also apply for $lim 1/f(x)= infty$ ? or $lim 1/f(x)= -infty$ If not then why?











share|cite|improve this question











$endgroup$








  • 3




    $begingroup$
    What if $f$ only takes negative values?
    $endgroup$
    – José Carlos Santos
    Dec 5 '18 at 11:21






  • 2




    $begingroup$
    the problem is whether you consider $infty$ to be either $pm infty$ or only $+infty$
    $endgroup$
    – TheD0ubleT
    Dec 5 '18 at 11:22












  • $begingroup$
    What if $f$ takes both positive and negative values ($f$ would not be continuous)?
    $endgroup$
    – Michael Burr
    Dec 5 '18 at 11:23










  • $begingroup$
    @MichaelBurr how can f take both of those, conjure to 0 and then not be continuous? when it's $lim_{xto a+}$?
    $endgroup$
    – Yuki1112
    Dec 5 '18 at 11:25












  • $begingroup$
    @TheD0ubleT either, sorry, I edited it to be more clear on the matter
    $endgroup$
    – Yuki1112
    Dec 5 '18 at 11:31














0












0








0





$begingroup$



Suppose that $f:DtoBbb R$, where $D$ is a subset of $Bbb R$ and $a$ is an accumulation point of $D$, $lim_{xto a+}f(x)=0$, and $f(x)ne0$ for any $x$ in $D$ in some neighborhood of $a$.
I understand from the following proof that if $lim f(x) = 0,$ then $lim 1/|f(x)| = infty.$ : proof for |f(x)|,
however, does it also apply for $lim 1/f(x)= infty$ ? or $lim 1/f(x)= -infty$ If not then why?











share|cite|improve this question











$endgroup$





Suppose that $f:DtoBbb R$, where $D$ is a subset of $Bbb R$ and $a$ is an accumulation point of $D$, $lim_{xto a+}f(x)=0$, and $f(x)ne0$ for any $x$ in $D$ in some neighborhood of $a$.
I understand from the following proof that if $lim f(x) = 0,$ then $lim 1/|f(x)| = infty.$ : proof for |f(x)|,
however, does it also apply for $lim 1/f(x)= infty$ ? or $lim 1/f(x)= -infty$ If not then why?








limits






share|cite|improve this question















share|cite|improve this question













share|cite|improve this question




share|cite|improve this question








edited Dec 5 '18 at 11:23







Yuki1112

















asked Dec 5 '18 at 11:18









Yuki1112Yuki1112

174




174








  • 3




    $begingroup$
    What if $f$ only takes negative values?
    $endgroup$
    – José Carlos Santos
    Dec 5 '18 at 11:21






  • 2




    $begingroup$
    the problem is whether you consider $infty$ to be either $pm infty$ or only $+infty$
    $endgroup$
    – TheD0ubleT
    Dec 5 '18 at 11:22












  • $begingroup$
    What if $f$ takes both positive and negative values ($f$ would not be continuous)?
    $endgroup$
    – Michael Burr
    Dec 5 '18 at 11:23










  • $begingroup$
    @MichaelBurr how can f take both of those, conjure to 0 and then not be continuous? when it's $lim_{xto a+}$?
    $endgroup$
    – Yuki1112
    Dec 5 '18 at 11:25












  • $begingroup$
    @TheD0ubleT either, sorry, I edited it to be more clear on the matter
    $endgroup$
    – Yuki1112
    Dec 5 '18 at 11:31














  • 3




    $begingroup$
    What if $f$ only takes negative values?
    $endgroup$
    – José Carlos Santos
    Dec 5 '18 at 11:21






  • 2




    $begingroup$
    the problem is whether you consider $infty$ to be either $pm infty$ or only $+infty$
    $endgroup$
    – TheD0ubleT
    Dec 5 '18 at 11:22












  • $begingroup$
    What if $f$ takes both positive and negative values ($f$ would not be continuous)?
    $endgroup$
    – Michael Burr
    Dec 5 '18 at 11:23










  • $begingroup$
    @MichaelBurr how can f take both of those, conjure to 0 and then not be continuous? when it's $lim_{xto a+}$?
    $endgroup$
    – Yuki1112
    Dec 5 '18 at 11:25












  • $begingroup$
    @TheD0ubleT either, sorry, I edited it to be more clear on the matter
    $endgroup$
    – Yuki1112
    Dec 5 '18 at 11:31








3




3




$begingroup$
What if $f$ only takes negative values?
$endgroup$
– José Carlos Santos
Dec 5 '18 at 11:21




$begingroup$
What if $f$ only takes negative values?
$endgroup$
– José Carlos Santos
Dec 5 '18 at 11:21




2




2




$begingroup$
the problem is whether you consider $infty$ to be either $pm infty$ or only $+infty$
$endgroup$
– TheD0ubleT
Dec 5 '18 at 11:22






$begingroup$
the problem is whether you consider $infty$ to be either $pm infty$ or only $+infty$
$endgroup$
– TheD0ubleT
Dec 5 '18 at 11:22














$begingroup$
What if $f$ takes both positive and negative values ($f$ would not be continuous)?
$endgroup$
– Michael Burr
Dec 5 '18 at 11:23




$begingroup$
What if $f$ takes both positive and negative values ($f$ would not be continuous)?
$endgroup$
– Michael Burr
Dec 5 '18 at 11:23












$begingroup$
@MichaelBurr how can f take both of those, conjure to 0 and then not be continuous? when it's $lim_{xto a+}$?
$endgroup$
– Yuki1112
Dec 5 '18 at 11:25






$begingroup$
@MichaelBurr how can f take both of those, conjure to 0 and then not be continuous? when it's $lim_{xto a+}$?
$endgroup$
– Yuki1112
Dec 5 '18 at 11:25














$begingroup$
@TheD0ubleT either, sorry, I edited it to be more clear on the matter
$endgroup$
– Yuki1112
Dec 5 '18 at 11:31




$begingroup$
@TheD0ubleT either, sorry, I edited it to be more clear on the matter
$endgroup$
– Yuki1112
Dec 5 '18 at 11:31










0






active

oldest

votes











Your Answer





StackExchange.ifUsing("editor", function () {
return StackExchange.using("mathjaxEditing", function () {
StackExchange.MarkdownEditor.creationCallbacks.add(function (editor, postfix) {
StackExchange.mathjaxEditing.prepareWmdForMathJax(editor, postfix, [["$", "$"], ["\\(","\\)"]]);
});
});
}, "mathjax-editing");

StackExchange.ready(function() {
var channelOptions = {
tags: "".split(" "),
id: "69"
};
initTagRenderer("".split(" "), "".split(" "), channelOptions);

StackExchange.using("externalEditor", function() {
// Have to fire editor after snippets, if snippets enabled
if (StackExchange.settings.snippets.snippetsEnabled) {
StackExchange.using("snippets", function() {
createEditor();
});
}
else {
createEditor();
}
});

function createEditor() {
StackExchange.prepareEditor({
heartbeatType: 'answer',
autoActivateHeartbeat: false,
convertImagesToLinks: true,
noModals: true,
showLowRepImageUploadWarning: true,
reputationToPostImages: 10,
bindNavPrevention: true,
postfix: "",
imageUploader: {
brandingHtml: "Powered by u003ca class="icon-imgur-white" href="https://imgur.com/"u003eu003c/au003e",
contentPolicyHtml: "User contributions licensed under u003ca href="https://creativecommons.org/licenses/by-sa/3.0/"u003ecc by-sa 3.0 with attribution requiredu003c/au003e u003ca href="https://stackoverflow.com/legal/content-policy"u003e(content policy)u003c/au003e",
allowUrls: true
},
noCode: true, onDemand: true,
discardSelector: ".discard-answer"
,immediatelyShowMarkdownHelp:true
});


}
});














draft saved

draft discarded


















StackExchange.ready(
function () {
StackExchange.openid.initPostLogin('.new-post-login', 'https%3a%2f%2fmath.stackexchange.com%2fquestions%2f3026953%2fif-lim-fx-0-then-lim-1-fx-infty%23new-answer', 'question_page');
}
);

Post as a guest















Required, but never shown

























0






active

oldest

votes








0






active

oldest

votes









active

oldest

votes






active

oldest

votes
















draft saved

draft discarded




















































Thanks for contributing an answer to Mathematics Stack Exchange!


  • Please be sure to answer the question. Provide details and share your research!

But avoid



  • Asking for help, clarification, or responding to other answers.

  • Making statements based on opinion; back them up with references or personal experience.


Use MathJax to format equations. MathJax reference.


To learn more, see our tips on writing great answers.




draft saved


draft discarded














StackExchange.ready(
function () {
StackExchange.openid.initPostLogin('.new-post-login', 'https%3a%2f%2fmath.stackexchange.com%2fquestions%2f3026953%2fif-lim-fx-0-then-lim-1-fx-infty%23new-answer', 'question_page');
}
);

Post as a guest















Required, but never shown





















































Required, but never shown














Required, but never shown












Required, but never shown







Required, but never shown

































Required, but never shown














Required, but never shown












Required, but never shown







Required, but never shown







Popular posts from this blog

Plaza Victoria

In PowerPoint, is there a keyboard shortcut for bulleted / numbered list?

How to put 3 figures in Latex with 2 figures side by side and 1 below these side by side images but in...